3

Suppose $X_1,\ldots,X_n$ are random samples of a $U(0,\theta)$ distribution. If $Y_n$ are the largest order statistics of the sample, and $V=\displaystyle\frac{n\bar{X}}{Y_n}$, how can I calculate the probability density function of $V$?

  • Duplicate of: http://math.stackexchange.com/questions/731089/calculating-var3t-y-n-in-uniform-distribution#comment1526812_731089 – wolfies Apr 01 '14 at 06:43
  • Duplicate of: http://stats.stackexchange.com/questions/91801/finding-var3t-y-n-in-uniform-distribution – wolfies Apr 01 '14 at 06:44
  • Duplicate of: http://stats.stackexchange.com/questions/91961/joint-distribution-of-a-random-variable-and-the-sample-maximum – wolfies Apr 01 '14 at 06:47
  • 1
    Definitely not a duplicate of 91961, because the joint distribution of $(X_1, Y_n)$ gives little information about $V$. – whuber Apr 01 '14 at 14:35

1 Answers1

4

Let the order statistics be $X_{[1]}\le X_{[2]}\le\cdots\le X_{[n]}$ and assume the $\{X_i\}$ are independent. The following are well known:

  1. The joint distribution of the order statistics $(X_{[i]})$ is uniform on the set $0\le X_{[1]}\le X_{[2]}\le\cdots\le X_{[n]}\le \theta.$ This is a consequence of (a) the uniformity of $(X_1,X_2,\ldots, X_n)$ and (b) the map that takes $(X_1, X_2, \ldots, X_n)$ into its sorted version (the order statistics) is an $n!$ to $1$ probability-preserving map. Thus the joint pdf of the order statistics is

    $$f_\theta(y_1,y_2, \ldots, y_n) = \frac{n!}{\theta^n}I(0\le y_1\le y_2\le \cdots \le y_n \le \theta)$$

    ($I$ is the indicator function).

  2. The marginal CDF of the maximum, $X_{[n]}$, is

    $$F_n(y_n) = {\Pr}_\theta(X_{[n]}\le y_n) = {\Pr}_\theta(X_1\le y_n)\cdots{\Pr}_\theta(X_n\le y_n) = \left(\frac{y_n}{\theta}\right)^n$$

    whence its PDF is

    $$f_n(y_n) = \frac{d F_n(y_n)}{d y_n} = \frac{n y_n^{n-1}}{\theta^n}.$$

Dividing $f_\theta$ by $f_n$ shows the conditional distribution of the sum

$$Y = X_{[1]}+\cdots X_{[n-1]} = X_1+X_2+\cdots+X_n - X_{[n]} = n\bar{X}-X_{[n]}$$

is

$$f_Y(y\ |\ X_{[n]}=y_n) = \frac{(n-1)!}{y_n^{n-1}}I(0\le y_1\le y_2\le \cdots \le y_{n-1} \le y_{n}).$$

This is recognizable (by comparing to result (1) above) as the distribution of the order statistics of $n-1$ iid uniform$(0, y_n)$ variates. Dividing by $y_n$ rescales them to iid uniform$(0,1)$ variates, whence the distribution of $\frac{Y}{X_{[n]}} = V-1$ is that of the sum of $n-1$ iid uniform$(0,1)$ variables. That distribution is described at elsewhere on our site at https://stats.stackexchange.com/a/43075, which gives four ways to derive its PDF.


An R simulation bears out this result. The output plots compare histograms of $V$ to its PDF (overdrawn in red); the match is excellent:

Histograms

#
# Compute the PDF of a sum of `n` iid uniform(0,1) variables.
#
unif.sum <- function(x, n) {
  theta <- function(y) ifelse(y >= 0, y^(n-1), 0)
  s <- outer(0:n, x, function(i, y) choose(n, i)*theta(y-i)*(-1)^i)
  apply(s, 2, sum) / factorial(n-1)
}
#
# Simulate V for various values of `n` and visually compare to the 
# theoretically derived PDF.
#
n.iter <- 10^5             # Number of samples per simulation
ns <- c(2, 3, 5, 12)       # Sample sizes to investigate
par(mfrow=c(1,length(ns))) # Display all plots in a row
for(n in ns) {
    V <- t(apply(matrix(runif(n*n.iter), ncol=n), 1, function(y) sum(y)/max(y)))
    hist(V, main=paste("n =", n), freq=FALSE)
    curve(unif.sum(x-1, n-1), col="Red", lwd=2, add=TRUE)
}
whuber
  • 322,774
  • Nice answer. But given that the same question has been posed by 3 different people (or 3 different names) on 2 different sites ... it seems you are probably doing their homework for them, which may not be a good thing. One can only hope that this arrived too late to be used in their assignment :) – wolfies Apr 02 '14 at 07:42
  • @wolfies This doesn't look like homework to me (for several reasons, including that the various questions reflect efforts to attack and simplify the problem). Regardless, we shouldn't care whether problems are homework. Policing student honesty is not our task. What matters for us are the nature and quality of the question: when a question is so obviously of a routine textbook nature we treat it a little differently than others. Although there's no clear dividing line between routine and non-routine, this one strikes me as being on the answerable side of that line. – whuber Apr 02 '14 at 17:12